Difference between revisions of "2008 AMC 8 Problems/Problem 14"

Line 17: Line 17:
 
==See Also==
 
==See Also==
 
{{AMC8 box|year=2008|num-b=13|num-a=15}}
 
{{AMC8 box|year=2008|num-b=13|num-a=15}}
 +
{{MAA Notice}}

Revision as of 01:38, 5 July 2013

Problem

Three $\text{A's}$, three $\text{B's}$, and three $\text{C's}$ are placed in the nine spaces so that each row and column contain one of each letter. If $\text{A}$ is placed in the upper left corner, how many arrangements are possible? [asy] size((80)); draw((0,0)--(9,0)--(9,9)--(0,9)--(0,0)); draw((3,0)--(3,9)); draw((6,0)--(6,9)); draw((0,3)--(9,3)); draw((0,6)--(9,6)); label("A", (1.5,7.5)); [/asy] $\textbf{(A)}\ 2\qquad\textbf{(B)}\ 3\qquad\textbf{(C)}\ 4\qquad\textbf{(D)}\ 5\qquad\textbf{(E)}\ 6$

Solution

There are $2$ ways to place the remaining $\text{A's}$, $2$ ways to place the remaining $\text{B's}$, and $1$ way to place the remaining $\text{C's}$ for a total of $(2)(2)(1) = \boxed{\textbf{(C)}\ 4}$.

See Also

2008 AMC 8 (ProblemsAnswer KeyResources)
Preceded by
Problem 13
Followed by
Problem 15
1 2 3 4 5 6 7 8 9 10 11 12 13 14 15 16 17 18 19 20 21 22 23 24 25
All AJHSME/AMC 8 Problems and Solutions

The problems on this page are copyrighted by the Mathematical Association of America's American Mathematics Competitions. AMC logo.png